Q12

 
nazu.s.shaikh
Thanks Received: 0
Elle Woods
Elle Woods
 
Posts: 53
Joined: April 27th, 2010
 
 
trophy
First Responder
 

Q12

by nazu.s.shaikh Mon Jul 05, 2010 7:15 pm

Question about this game regarding the constraints, when answering number 12 I noticed that Q would be present but not F .. K would be present and G wouldn't.

At first glance I assumed the constraints meant that If F was present Q needed to be present too but when looking at the constraints again in regards to answering Q12 having K present doesn't mean G has to be there.. is that because of the word "If" there?

And Q15...why is B the answer and not A... having K present in the committee would mean that (if G were not chosen) then you'd have G/J/M and you can't have J or M because there are not enough spaces for J & M.

Which lead me to choose A (G) as my answer. Having G in the committee would force K to be there.

Lastly (sorry), Q19 the question asks " at least one" ...I must be misinterpreting the question because each of the answer choices work for me if I have at least just 1 of the representatives from the pair? What am I doing wrong here?
User avatar
 
ManhattanPrepLSAT1
Thanks Received: 1909
Atticus Finch
Atticus Finch
 
Posts: 2851
Joined: October 07th, 2009
 
 
 

Re: PT11, S1, G3 - A housing committee will consist

by ManhattanPrepLSAT1 Mon Jul 05, 2010 10:42 pm

Really good questions, and ones that I think as you continue to prepare for the LSAT, you'll find are at the heart of the test.

Let's start with the 3rd and 4th constraints.

3. If F is selected, then Q must be selected.
4. If G is selected, then K must be selected.

Both of these constraints do not imply that F and Q are selected together and that G and K are selected together. Instead the "if" part implies the "then" part. So, if F is selected then Q must be, but if F is not selected, then we do not know anything about Q. Likewise, if G is selected then K must be selected, but if G is not selected, then we do not know anything about K.

15. If F is the chairperson of the committee

_ _ / _ _ _
tenets/homeowners

we can get a sense for how many tenets and how many homeowners from the second constraint.

If F is selected then

F _ / _ _ _
tenets/homeowners

We could not select J, M, nor G because each one of those would require that we have too many tenets. If J or M were selected we would need both, but we do not have room for two more tenets. Also, if G were selected, then we would need to select K, but again we do not have room for two more tenets. Since we can rule out J, M and G, the only tenet left that can be selected is K - answer choice (B).

19. The committee must include at least one representative from which one of the following pairs.

Answer choice (A) could be eliminated if you could find a hypothetical in which neither F not P were selected. So for each answer choice, see if you can have neither of the characters mentioned. For four of the answer choices you should be able to find solutions without any of the characters mentioned in the pairs. But for one for one of the pairs, you will not be able to find a hypothetical that satisfies all of the constraints with neither character - that will be the correct answer.

I'll let you try and work through question 19 on your own, but if you need help, let me know and I'll walk you through it!